For what $n$ is $U_n$ cyclic?












26












$begingroup$



When can we say a multiplicative group of integers modulo $n$, i.e., $U_n$ is cyclic?




$$U_n={a inmathbb Z_n mid gcd(a,n)=1 }$$



I searched the internet but did not get a clear idea.










share|cite|improve this question











$endgroup$

















    26












    $begingroup$



    When can we say a multiplicative group of integers modulo $n$, i.e., $U_n$ is cyclic?




    $$U_n={a inmathbb Z_n mid gcd(a,n)=1 }$$



    I searched the internet but did not get a clear idea.










    share|cite|improve this question











    $endgroup$















      26












      26








      26


      19



      $begingroup$



      When can we say a multiplicative group of integers modulo $n$, i.e., $U_n$ is cyclic?




      $$U_n={a inmathbb Z_n mid gcd(a,n)=1 }$$



      I searched the internet but did not get a clear idea.










      share|cite|improve this question











      $endgroup$





      When can we say a multiplicative group of integers modulo $n$, i.e., $U_n$ is cyclic?




      $$U_n={a inmathbb Z_n mid gcd(a,n)=1 }$$



      I searched the internet but did not get a clear idea.







      abstract-algebra group-theory cyclic-groups






      share|cite|improve this question















      share|cite|improve this question













      share|cite|improve this question




      share|cite|improve this question








      edited Feb 7 '16 at 11:17









      Rudy the Reindeer

      26.8k1796245




      26.8k1796245










      asked Feb 26 '13 at 12:58









      SankhaSankha

      595722




      595722






















          3 Answers
          3






          active

          oldest

          votes


















          26












          $begingroup$

          So $U_n$ is the group of units in $mathbb{Z}/nmathbb{Z}$.



          Write the prime decomposition
          $$
          n=p_1^{alpha_1}cdots p_r^{alpha_r}.
          $$



          By the Chinese remainder theorem
          $$
          mathbb{Z}/nmathbb{Z}=mathbb{Z}/p_1^{alpha_1}mathbb{Z}timesldotstimesmathbb{Z}/p_r^{alpha_r}mathbb{Z}
          $$
          so
          $$
          U_n=U_{p_1^{alpha_1}}timesldotstimes U_{p_r^{alpha_r}}.
          $$



          For powers of $2$, we have
          $$
          U_2={0}
          $$
          and for $kgeq 2$
          $$
          U_{2^k}=mathbb{Z}/2mathbb{Z}times mathbb{Z}/2^{k-2}mathbb{Z}.
          $$



          For odd primes $p$,
          $$
          U_{p^alpha}=mathbb{Z}/phi(p^alpha)mathbb{Z}=mathbb{Z}/p^{alpha-1}(p-1)mathbb{Z}.
          $$



          So you see now that $U_n$ is cyclic if and only if
          $$
          n=2,4,p^alpha,2p^{alpha}
          $$
          where $p$ is an odd prime.



          Here is a reference.






          share|cite|improve this answer











          $endgroup$









          • 2




            $begingroup$
            Why is it true that $U_{p^k}=mathbb{Z}/2mathbb{Z}timesmathbb{Z}/2^{k-2}mathbb{Z}$?
            $endgroup$
            – Rasputin
            Jan 20 '17 at 20:03










          • $begingroup$
            Julien, why doesn't the even prime work please?
            $endgroup$
            – BCLC
            Oct 17 '18 at 11:48



















          10












          $begingroup$

          $U_n$ is cyclic iff $n$ is $2$, $4$, $p^k$, or $2p^k$, where $p$ is an odd prime.



          The proof follows from the Chinese Remainder Theorem for rings and the fact that $C_m times C_n$ is cyclic iff $(m,n)=1$ (here $C_n$ is the cyclic group of order $n$).



          The hard part is proving that $U_p$ is cyclic and this follows from the fact that $mathbb Z/p$ is a field and that $n = sum_{dmid n} phi(d)$.



          Any book on elementary number theory has a proof of this theorem. See for instance André Weil's Number theory for beginners, Leveque's Fundamentals of Number Theory, and Bolker's Elementary Number Theory.






          share|cite|improve this answer











          $endgroup$





















            5












            $begingroup$

            Here "cyclic if and only if $varphi(n)=lambda(n)$" but there's no proof - the proof is elementary but very tricky.






            share|cite|improve this answer











            $endgroup$














              Your Answer





              StackExchange.ifUsing("editor", function () {
              return StackExchange.using("mathjaxEditing", function () {
              StackExchange.MarkdownEditor.creationCallbacks.add(function (editor, postfix) {
              StackExchange.mathjaxEditing.prepareWmdForMathJax(editor, postfix, [["$", "$"], ["\\(","\\)"]]);
              });
              });
              }, "mathjax-editing");

              StackExchange.ready(function() {
              var channelOptions = {
              tags: "".split(" "),
              id: "69"
              };
              initTagRenderer("".split(" "), "".split(" "), channelOptions);

              StackExchange.using("externalEditor", function() {
              // Have to fire editor after snippets, if snippets enabled
              if (StackExchange.settings.snippets.snippetsEnabled) {
              StackExchange.using("snippets", function() {
              createEditor();
              });
              }
              else {
              createEditor();
              }
              });

              function createEditor() {
              StackExchange.prepareEditor({
              heartbeatType: 'answer',
              autoActivateHeartbeat: false,
              convertImagesToLinks: true,
              noModals: true,
              showLowRepImageUploadWarning: true,
              reputationToPostImages: 10,
              bindNavPrevention: true,
              postfix: "",
              imageUploader: {
              brandingHtml: "Powered by u003ca class="icon-imgur-white" href="https://imgur.com/"u003eu003c/au003e",
              contentPolicyHtml: "User contributions licensed under u003ca href="https://creativecommons.org/licenses/by-sa/3.0/"u003ecc by-sa 3.0 with attribution requiredu003c/au003e u003ca href="https://stackoverflow.com/legal/content-policy"u003e(content policy)u003c/au003e",
              allowUrls: true
              },
              noCode: true, onDemand: true,
              discardSelector: ".discard-answer"
              ,immediatelyShowMarkdownHelp:true
              });


              }
              });














              draft saved

              draft discarded


















              StackExchange.ready(
              function () {
              StackExchange.openid.initPostLogin('.new-post-login', 'https%3a%2f%2fmath.stackexchange.com%2fquestions%2f314846%2ffor-what-n-is-u-n-cyclic%23new-answer', 'question_page');
              }
              );

              Post as a guest















              Required, but never shown

























              3 Answers
              3






              active

              oldest

              votes








              3 Answers
              3






              active

              oldest

              votes









              active

              oldest

              votes






              active

              oldest

              votes









              26












              $begingroup$

              So $U_n$ is the group of units in $mathbb{Z}/nmathbb{Z}$.



              Write the prime decomposition
              $$
              n=p_1^{alpha_1}cdots p_r^{alpha_r}.
              $$



              By the Chinese remainder theorem
              $$
              mathbb{Z}/nmathbb{Z}=mathbb{Z}/p_1^{alpha_1}mathbb{Z}timesldotstimesmathbb{Z}/p_r^{alpha_r}mathbb{Z}
              $$
              so
              $$
              U_n=U_{p_1^{alpha_1}}timesldotstimes U_{p_r^{alpha_r}}.
              $$



              For powers of $2$, we have
              $$
              U_2={0}
              $$
              and for $kgeq 2$
              $$
              U_{2^k}=mathbb{Z}/2mathbb{Z}times mathbb{Z}/2^{k-2}mathbb{Z}.
              $$



              For odd primes $p$,
              $$
              U_{p^alpha}=mathbb{Z}/phi(p^alpha)mathbb{Z}=mathbb{Z}/p^{alpha-1}(p-1)mathbb{Z}.
              $$



              So you see now that $U_n$ is cyclic if and only if
              $$
              n=2,4,p^alpha,2p^{alpha}
              $$
              where $p$ is an odd prime.



              Here is a reference.






              share|cite|improve this answer











              $endgroup$









              • 2




                $begingroup$
                Why is it true that $U_{p^k}=mathbb{Z}/2mathbb{Z}timesmathbb{Z}/2^{k-2}mathbb{Z}$?
                $endgroup$
                – Rasputin
                Jan 20 '17 at 20:03










              • $begingroup$
                Julien, why doesn't the even prime work please?
                $endgroup$
                – BCLC
                Oct 17 '18 at 11:48
















              26












              $begingroup$

              So $U_n$ is the group of units in $mathbb{Z}/nmathbb{Z}$.



              Write the prime decomposition
              $$
              n=p_1^{alpha_1}cdots p_r^{alpha_r}.
              $$



              By the Chinese remainder theorem
              $$
              mathbb{Z}/nmathbb{Z}=mathbb{Z}/p_1^{alpha_1}mathbb{Z}timesldotstimesmathbb{Z}/p_r^{alpha_r}mathbb{Z}
              $$
              so
              $$
              U_n=U_{p_1^{alpha_1}}timesldotstimes U_{p_r^{alpha_r}}.
              $$



              For powers of $2$, we have
              $$
              U_2={0}
              $$
              and for $kgeq 2$
              $$
              U_{2^k}=mathbb{Z}/2mathbb{Z}times mathbb{Z}/2^{k-2}mathbb{Z}.
              $$



              For odd primes $p$,
              $$
              U_{p^alpha}=mathbb{Z}/phi(p^alpha)mathbb{Z}=mathbb{Z}/p^{alpha-1}(p-1)mathbb{Z}.
              $$



              So you see now that $U_n$ is cyclic if and only if
              $$
              n=2,4,p^alpha,2p^{alpha}
              $$
              where $p$ is an odd prime.



              Here is a reference.






              share|cite|improve this answer











              $endgroup$









              • 2




                $begingroup$
                Why is it true that $U_{p^k}=mathbb{Z}/2mathbb{Z}timesmathbb{Z}/2^{k-2}mathbb{Z}$?
                $endgroup$
                – Rasputin
                Jan 20 '17 at 20:03










              • $begingroup$
                Julien, why doesn't the even prime work please?
                $endgroup$
                – BCLC
                Oct 17 '18 at 11:48














              26












              26








              26





              $begingroup$

              So $U_n$ is the group of units in $mathbb{Z}/nmathbb{Z}$.



              Write the prime decomposition
              $$
              n=p_1^{alpha_1}cdots p_r^{alpha_r}.
              $$



              By the Chinese remainder theorem
              $$
              mathbb{Z}/nmathbb{Z}=mathbb{Z}/p_1^{alpha_1}mathbb{Z}timesldotstimesmathbb{Z}/p_r^{alpha_r}mathbb{Z}
              $$
              so
              $$
              U_n=U_{p_1^{alpha_1}}timesldotstimes U_{p_r^{alpha_r}}.
              $$



              For powers of $2$, we have
              $$
              U_2={0}
              $$
              and for $kgeq 2$
              $$
              U_{2^k}=mathbb{Z}/2mathbb{Z}times mathbb{Z}/2^{k-2}mathbb{Z}.
              $$



              For odd primes $p$,
              $$
              U_{p^alpha}=mathbb{Z}/phi(p^alpha)mathbb{Z}=mathbb{Z}/p^{alpha-1}(p-1)mathbb{Z}.
              $$



              So you see now that $U_n$ is cyclic if and only if
              $$
              n=2,4,p^alpha,2p^{alpha}
              $$
              where $p$ is an odd prime.



              Here is a reference.






              share|cite|improve this answer











              $endgroup$



              So $U_n$ is the group of units in $mathbb{Z}/nmathbb{Z}$.



              Write the prime decomposition
              $$
              n=p_1^{alpha_1}cdots p_r^{alpha_r}.
              $$



              By the Chinese remainder theorem
              $$
              mathbb{Z}/nmathbb{Z}=mathbb{Z}/p_1^{alpha_1}mathbb{Z}timesldotstimesmathbb{Z}/p_r^{alpha_r}mathbb{Z}
              $$
              so
              $$
              U_n=U_{p_1^{alpha_1}}timesldotstimes U_{p_r^{alpha_r}}.
              $$



              For powers of $2$, we have
              $$
              U_2={0}
              $$
              and for $kgeq 2$
              $$
              U_{2^k}=mathbb{Z}/2mathbb{Z}times mathbb{Z}/2^{k-2}mathbb{Z}.
              $$



              For odd primes $p$,
              $$
              U_{p^alpha}=mathbb{Z}/phi(p^alpha)mathbb{Z}=mathbb{Z}/p^{alpha-1}(p-1)mathbb{Z}.
              $$



              So you see now that $U_n$ is cyclic if and only if
              $$
              n=2,4,p^alpha,2p^{alpha}
              $$
              where $p$ is an odd prime.



              Here is a reference.







              share|cite|improve this answer














              share|cite|improve this answer



              share|cite|improve this answer








              edited Mar 18 '16 at 21:25









              user26857

              39.5k124284




              39.5k124284










              answered Feb 26 '13 at 13:24









              JulienJulien

              38.8k358131




              38.8k358131








              • 2




                $begingroup$
                Why is it true that $U_{p^k}=mathbb{Z}/2mathbb{Z}timesmathbb{Z}/2^{k-2}mathbb{Z}$?
                $endgroup$
                – Rasputin
                Jan 20 '17 at 20:03










              • $begingroup$
                Julien, why doesn't the even prime work please?
                $endgroup$
                – BCLC
                Oct 17 '18 at 11:48














              • 2




                $begingroup$
                Why is it true that $U_{p^k}=mathbb{Z}/2mathbb{Z}timesmathbb{Z}/2^{k-2}mathbb{Z}$?
                $endgroup$
                – Rasputin
                Jan 20 '17 at 20:03










              • $begingroup$
                Julien, why doesn't the even prime work please?
                $endgroup$
                – BCLC
                Oct 17 '18 at 11:48








              2




              2




              $begingroup$
              Why is it true that $U_{p^k}=mathbb{Z}/2mathbb{Z}timesmathbb{Z}/2^{k-2}mathbb{Z}$?
              $endgroup$
              – Rasputin
              Jan 20 '17 at 20:03




              $begingroup$
              Why is it true that $U_{p^k}=mathbb{Z}/2mathbb{Z}timesmathbb{Z}/2^{k-2}mathbb{Z}$?
              $endgroup$
              – Rasputin
              Jan 20 '17 at 20:03












              $begingroup$
              Julien, why doesn't the even prime work please?
              $endgroup$
              – BCLC
              Oct 17 '18 at 11:48




              $begingroup$
              Julien, why doesn't the even prime work please?
              $endgroup$
              – BCLC
              Oct 17 '18 at 11:48











              10












              $begingroup$

              $U_n$ is cyclic iff $n$ is $2$, $4$, $p^k$, or $2p^k$, where $p$ is an odd prime.



              The proof follows from the Chinese Remainder Theorem for rings and the fact that $C_m times C_n$ is cyclic iff $(m,n)=1$ (here $C_n$ is the cyclic group of order $n$).



              The hard part is proving that $U_p$ is cyclic and this follows from the fact that $mathbb Z/p$ is a field and that $n = sum_{dmid n} phi(d)$.



              Any book on elementary number theory has a proof of this theorem. See for instance André Weil's Number theory for beginners, Leveque's Fundamentals of Number Theory, and Bolker's Elementary Number Theory.






              share|cite|improve this answer











              $endgroup$


















                10












                $begingroup$

                $U_n$ is cyclic iff $n$ is $2$, $4$, $p^k$, or $2p^k$, where $p$ is an odd prime.



                The proof follows from the Chinese Remainder Theorem for rings and the fact that $C_m times C_n$ is cyclic iff $(m,n)=1$ (here $C_n$ is the cyclic group of order $n$).



                The hard part is proving that $U_p$ is cyclic and this follows from the fact that $mathbb Z/p$ is a field and that $n = sum_{dmid n} phi(d)$.



                Any book on elementary number theory has a proof of this theorem. See for instance André Weil's Number theory for beginners, Leveque's Fundamentals of Number Theory, and Bolker's Elementary Number Theory.






                share|cite|improve this answer











                $endgroup$
















                  10












                  10








                  10





                  $begingroup$

                  $U_n$ is cyclic iff $n$ is $2$, $4$, $p^k$, or $2p^k$, where $p$ is an odd prime.



                  The proof follows from the Chinese Remainder Theorem for rings and the fact that $C_m times C_n$ is cyclic iff $(m,n)=1$ (here $C_n$ is the cyclic group of order $n$).



                  The hard part is proving that $U_p$ is cyclic and this follows from the fact that $mathbb Z/p$ is a field and that $n = sum_{dmid n} phi(d)$.



                  Any book on elementary number theory has a proof of this theorem. See for instance André Weil's Number theory for beginners, Leveque's Fundamentals of Number Theory, and Bolker's Elementary Number Theory.






                  share|cite|improve this answer











                  $endgroup$



                  $U_n$ is cyclic iff $n$ is $2$, $4$, $p^k$, or $2p^k$, where $p$ is an odd prime.



                  The proof follows from the Chinese Remainder Theorem for rings and the fact that $C_m times C_n$ is cyclic iff $(m,n)=1$ (here $C_n$ is the cyclic group of order $n$).



                  The hard part is proving that $U_p$ is cyclic and this follows from the fact that $mathbb Z/p$ is a field and that $n = sum_{dmid n} phi(d)$.



                  Any book on elementary number theory has a proof of this theorem. See for instance André Weil's Number theory for beginners, Leveque's Fundamentals of Number Theory, and Bolker's Elementary Number Theory.







                  share|cite|improve this answer














                  share|cite|improve this answer



                  share|cite|improve this answer








                  edited Feb 26 '13 at 14:59









                  Michael Hardy

                  1




                  1










                  answered Feb 26 '13 at 13:11









                  lhflhf

                  167k11172404




                  167k11172404























                      5












                      $begingroup$

                      Here "cyclic if and only if $varphi(n)=lambda(n)$" but there's no proof - the proof is elementary but very tricky.






                      share|cite|improve this answer











                      $endgroup$


















                        5












                        $begingroup$

                        Here "cyclic if and only if $varphi(n)=lambda(n)$" but there's no proof - the proof is elementary but very tricky.






                        share|cite|improve this answer











                        $endgroup$
















                          5












                          5








                          5





                          $begingroup$

                          Here "cyclic if and only if $varphi(n)=lambda(n)$" but there's no proof - the proof is elementary but very tricky.






                          share|cite|improve this answer











                          $endgroup$



                          Here "cyclic if and only if $varphi(n)=lambda(n)$" but there's no proof - the proof is elementary but very tricky.







                          share|cite|improve this answer














                          share|cite|improve this answer



                          share|cite|improve this answer








                          edited Dec 16 '13 at 9:18

























                          answered Feb 26 '13 at 13:07







                          user58512





































                              draft saved

                              draft discarded




















































                              Thanks for contributing an answer to Mathematics Stack Exchange!


                              • Please be sure to answer the question. Provide details and share your research!

                              But avoid



                              • Asking for help, clarification, or responding to other answers.

                              • Making statements based on opinion; back them up with references or personal experience.


                              Use MathJax to format equations. MathJax reference.


                              To learn more, see our tips on writing great answers.




                              draft saved


                              draft discarded














                              StackExchange.ready(
                              function () {
                              StackExchange.openid.initPostLogin('.new-post-login', 'https%3a%2f%2fmath.stackexchange.com%2fquestions%2f314846%2ffor-what-n-is-u-n-cyclic%23new-answer', 'question_page');
                              }
                              );

                              Post as a guest















                              Required, but never shown





















































                              Required, but never shown














                              Required, but never shown












                              Required, but never shown







                              Required, but never shown

































                              Required, but never shown














                              Required, but never shown












                              Required, but never shown







                              Required, but never shown







                              Popular posts from this blog

                              Biblatex bibliography style without URLs when DOI exists (in Overleaf with Zotero bibliography)

                              ComboBox Display Member on multiple fields

                              Is it possible to collect Nectar points via Trainline?